Загрузка страницы

The Most Intimidating Integral I've Ever Seen

This Putnam Series was given on the Putnam exam in 1997 (Problem A3). Let's figure out a way to deal with this Putnam problem!

For those that don't know, the Putnam math competition features some super interesting and challenging problems!

Some of the links below are affiliate links. As an Amazon Associate I earn from qualifying purchases. If you purchase through these links, it won't cost you any additional cash, but it will help to support my channel. Thank you!

►BECOME A CHANNEL MEMBER
https://www.youtube.com/channel/UChVUSXFzV8QCOKNWGfE56YQ/join
►STUFF I RECOMMEND
https://www.amazon.com/shop/brithemathguy (Affiliate Links)

Putnam 1997 A3
Putnam Exam 1997 A3

#math #brithemathguy #putnam

Видео The Most Intimidating Integral I've Ever Seen канала BriTheMathGuy
Показать
Комментарии отсутствуют
Введите заголовок:

Введите адрес ссылки:

Введите адрес видео с YouTube:

Зарегистрируйтесь или войдите с
Информация о видео
28 июня 2021 г. 17:00:18
00:06:36
Яндекс.Метрика